Contracts Flashcards

1
Q

Opal offered, in writing, to sell Larkin a parcel of land for $300,000. If Opal dies, the offer will:

  • Terminate prior to Larkin’s acceptance only if Larkin received notice of Opal’s death.
  • Remain open for a reasonable period of time after Opal’s death.
  • Automatically terminate despite Larkin’s prior acceptance.
  • Automatically terminate prior to Larkin’s acceptance.
A

Automatically terminate prior to Larkin’s acceptance. An offer, unless irrevocable, IS terminated immediately upon the death of the offeror, and thus no contract could be formed.

C would be correct if there had been a valid acceptance first, a contract would have been formed. If there is a CONTRACT, then death does not always terminate contract obligations.

How well did you know this?
1
Not at all
2
3
4
5
Perfectly
2
Q

On September 27, Summers sent Fox a letter offering to sell Fox a vacation home for $150,000. On October 2, Fox replied by mail agreeing to buy the home for $145,000. Summers did not reply to Fox.

Do Fox and Summers have a binding contract?

  • No, because Fox failed to sign and return Summers’ letter.
  • No, because Fox’s letter was a counteroffer.
  • Yes, because Summers’ offer was validly accepted.
  • Yes, because Summers’ silence is an implied acceptance of Fox’s letter.
A

No, because Fox’s letter was a counteroffer. A counteroffer rejects the original offer. This is a counteroffer, because the terms differ from those in the original offer. For a contract involving real estate or almost anything other than a sale of goods, the “mirror image” rule is in place. If the response is not a mirror image acceptance of the offer, then it is a rejection and if accompanied by a new offer, a counteroffer.

How well did you know this?
1
Not at all
2
3
4
5
Perfectly
3
Q

On April 1, Fine Corp. faxed Moss an offer to purchase Moss’ warehouse for $500,000. The offer stated that it would remain open only until April 4 and that acceptance must be received to be effective. Moss sent an acceptance on April 4 by overnight mail and Fine received it on April 5.

Which of the following statements is correct?

  • No contract was formed because Moss sent the acceptance by an unauthorized method.
  • No contract was formed because Fine received Moss’ acceptance after April 4.
  • A contract was formed when Moss sent the acceptance.
  • A contract was formed when Fine received Moss’ acceptance.​
A

No contract was formed because Fine received Moss’ acceptance after April 4. Although most acceptances of bilateral offers are sent by an authorized medium and effective when sent by the authorized medium, the offeror can condition acceptance to not be effective until received. Therefore, regardless of the medium used, the acceptance must be received before the offer terminates by lapse of time. This offer terminated at midnight on April 4, and the acceptance was not received until April 5, after the offer was terminated.

How well did you know this?
1
Not at all
2
3
4
5
Perfectly
4
Q

Covenants Not to Compete

A sale of a business containing a covenant prohibiting seller from owning or operating similar business as well as the termination of an employee who has agreed not to compete are legal and enforceable provided the agreement:

A
  • Protects legitimate interests of buyer or employer without creating too large a burden on seller or employee (based on ability to find other work)
  • Is reasonable as to length of time under the circumstances to protect those interests
  • Is reasonable as to area to protect interests of same area
  • Same whether employer or employee initiated termination
How well did you know this?
1
Not at all
2
3
4
5
Perfectly
5
Q

Wert, an employee of Salam Corp., signed an agreement not to compete with Salam during and after being employed with Salam. Wert is the director of research and has knowledge of many of Salam’s trade secrets. If Wert’s employment with Salam is terminated and Wert wishes to compete with Salam, which of the following statements is not correct?

  • The agreement is only enforceable if Wert voluntarily terminates his employment with Salam.
  • The agreement must be necessary to protect Salam’s legitimate interests in order to be enforceable.
  • The geographic area covered by the agreement must be reasonable in order to be enforceable.
  • The court will consider Wert’s ability to obtain other employment against Salam’s right to protect its business.
A

The agreement is only enforceable if Wert voluntarily terminates his employment with Salam.

When deciding whether a covenant not to compete is enforceable, it is irrelevant whether the employee voluntarily terminates his employment. To be enforceable, the agreement must protect Salam’s legitimate interests and the geographic area covered must be reasonable. In deciding the issue, the court will balance Wert’s ability to obtain other employment against Salam’s right to protect its business.

How well did you know this?
1
Not at all
2
3
4
5
Perfectly
6
Q

Undue Influence

A

Unfair persuasion of one person over another which prevents understanding or voluntary action

  1. Usually occurs when very dominant person has extreme influence over weaker person
  2. Also occurs through abuse of fiduciary relationship (e.g., CPA, attorney, guardian, trustee, etc.)
  3. Normally causes agreement to be voidable
How well did you know this?
1
Not at all
2
3
4
5
Perfectly
7
Q

To satisfy the UCC Statute of Frauds, a written agreement for the sale of goods must

  • Contain payment terms.
  • Be signed by both buyer and seller.
  • Indicate that a contract for sale has been made.
  • Refer to the time and place of delivery.
A

Indicate that a contract for sale has been made. Under the UCC Statute of Frauds, a written agreement for the sale of goods is adequate if it indicates a contract for the sale of goods has been made between parties and is signed by the party to be charged. The written agreement may omit material terms (i.e., price, delivery, time for performance) as long as the quantity is stated. Reasonable terms will be inferred for those terms which are missing from the written agreement.

How well did you know this?
1
Not at all
2
3
4
5
Perfectly
8
Q

Dunne and Cook signed a contract requiring Cook to rebind 500 of Dunne’s books at $3.00 per book. Later, Dunne requested, in good faith, that the price be reduced to $2.70 per book. Cook agreed orally to reduce the price to $2.70.

Under the circumstances, the oral agreement is

  • Enforceable, but proof of it is inadmissible into evidence.
  • Enforceable, and proof of it is admissible into evidence.
  • Unenforceable, because Dunne failed to give consideration, but proof of it is otherwise admissible into evidence.
  • Unenforceable, due to the statute of frauds, and proof of it is inadmissible into evidence.
A

Unenforceable, because Dunne failed to give consideration, but proof of it is otherwise admissible into evidence. There are two issues here.

  1. The first is the enforceability of the modification. This modification is invalid, because it is a contract for services, and it is not supported by new consideration. Dunne has not agreed to do anything new, and, therefore, since he had a preexisting duty to pay $3.00 per book, the modification cannot be enforced.
  2. The second issue is whether evidence of the agreement can be introduced for other reasons. The answer here is yes. The parole evidence rule prohibits testimony about agreements that existed before a contract was signed, because contracts are generally presumed to be the final word on an agreement. However, evidence about things taking place after a contact was signed (subsequent modification) is admissible.
How well did you know this?
1
Not at all
2
3
4
5
Perfectly
9
Q

In determining whether the consideration requirement to form a contract has been satisfied, the consideration exchanged by the parties to the contract must be:

  • Of approximately equal value.
  • Legally sufficient.
  • Exchanged simultaneously by the parties.
  • Fair and reasonable under the circumstances.
A

Legally sufficient. Consideration must be sufficient. However, the general rule is that any obligation of legal value and bargained-for is sufficient consideration.

How well did you know this?
1
Not at all
2
3
4
5
Perfectly
10
Q

In which of the following situations does the first promise serve as valid consideration for the second promise?

  • A police officer’s promise to catch a thief for a victim’s promise to pay a reward.
  • A builder’s promise to complete a contract for a purchaser’s promise to extend the time for completion.
  • A debtor’s promise to pay $500 for a creditor’s promise to forgive the balance of a $600 liquidated debt.
  • A debtor’s promise to pay $500 for a creditor’s promise to forgive the balance of a $600 disputed debt.
A

A debtor’s promise to pay $500 for a creditor’s promise to forgive the balance of a $600 disputed debt. To give consideration, a person must promise to do something new or something not already obligated to do. If the debt is rightfully disputed, a debtor is not under a preexisting obligation to pay the full amount. In offering to pay $500, the offeror is promising to do something he or she does not otherwise have to do – pay $500. This promise, then, is consideration that supports the forgiveness of the rest of the disputed debt.

How well did you know this?
1
Not at all
2
3
4
5
Perfectly
11
Q

Define Consideration

A

Consideration: Consists of the benefit promised by the offeror (promisor) and the legal detriment promised or performed by the offeree (promisee). In a bilateral contract, both the offeror and offeree are promisors (those making a promise) and promisees (those receiving the promise). Both sides must have benefit and detriment for valid consideration to be present and the detriment on one side induces the detriment on the other side. The exchange of the detriment is bargained for by the parties.

How well did you know this?
1
Not at all
2
3
4
5
Perfectly
12
Q

Fiore owed Lutz $5,000. As the result of an unrelated transaction, Lutz owed Bing that same amount. The three parties signed an agreement that Fiore would pay Bing instead of Lutz and Lutz would be discharged from all liability. The agreement among the parties is

  • Unenforceable for lack of consideration.
  • Voidable at Bing’s option.
  • An executed accord and satisfaction.
  • A novation.
A

A novation. A novation is a three-party agreement between the contracting parties and a third party, whereby one of the contracting parties is discharged from his/her duty and the third party is substituted in the discharged party’s place. In this case, all three parties agree to discharge the old contracts between Fiore and Lutz, and Lutz and Bing, by the creation of a new contract between Fiore and Bing. The new contract is a novation.

How well did you know this?
1
Not at all
2
3
4
5
Perfectly
13
Q

A party to a contract who seeks to rescind the contract because of that party’s reliance on the unintentional but materially false statements of the other party will assert

  • Reformation.
  • Actual fraud.
  • Misrepresentation.
  • Constructive fraud.
A

Misrepresentation. may involve an innocent misstatement made in good faith (i.e., there is no scienter or intent to mislead). In order to rescind a contract because of a misrepresentation, the rescinding party must prove that there was a misrepresentation of a material fact, that there was reliance on this fact, and that there was injury as a result.

How well did you know this?
1
Not at all
2
3
4
5
Perfectly
14
Q

Breach of Contract Statute of Limitations

A

Statute begins to run from time cause of action accrues (e.g., breach)

How well did you know this?
1
Not at all
2
3
4
5
Perfectly
15
Q

With regard to an agreement for the sale of real estate, the Statute of Frauds

  • Does not require that the agreement be signed by all parties.
  • Does not apply if the value of the real estate is less than $500.
  • Requires that the entire agreement be in a single writing.
  • Requires that the purchase price be fair and adequate in relation to the value of the real estate.
A

Does not require that the agreement be signed by all parties.

Contracts required to be in writing and signed by party to be charged—these are said to be within the Statute:

  1. An agreement to sell land or any interest in land
    1. Includes buildings, easements, and contracts to sell real estate
    2. Part performance typically satisfies Statute even though real estate contract was oral, but this requires
      1. Possession of the land
      2. Either part payment or making of improvements on real estate
      3. Many courts require all three
How well did you know this?
1
Not at all
2
3
4
5
Perfectly
16
Q

All of the following statements regarding compliance with the statute of frauds are correct except:

  • Any necessary writing must be signed by all parties against whom enforcement is sought.
  • Contracts involving the sale of goods in an amount greater than $500 must be in writing.
  • Contract terms must be contained in only one document.
  • Contracts that by their terms cannot be completed within one year must be in writing.
A

Contract terms must be contained in only one document.

How well did you know this?
1
Not at all
2
3
4
5
Perfectly
17
Q

Statute of Frauds - Exception to Writing Requirement

A
  1. Oral contract involving specially manufactured goods (i.e., not saleable in ordinary course of business) if seller has made substantial start in their manufacture (or even made a contract for necessary raw materials) is enforceable
  2. Oral contract is enforceable against party who admits it in court but not beyond quantity of goods admitted
  3. Goods that have been paid for (if seller accepts payment) or goods which buyer has accepted are part of enforceable contract even if oral
How well did you know this?
1
Not at all
2
3
4
5
Perfectly
18
Q

Kent, a 16-year old, purchased a used car from Mint Motors, Inc. Ten months later, the car was stolen and never recovered. Which of the following statements is correct?

  • The car’s theft is a de facto ratification of the purchase because it is impossible to return the car.
  • Kent may disaffirm the purchase because Kent is a minor.
  • Kent effectively ratified the purchase because Kent used the car for an unreasonable period of time.
  • Kent may disaffirm the purchase because Mint, a merchant, is subject to the UCC.
A

Kent may disaffirm the purchase because Kent is a minor. Minor may disaffirm contract at any time until a reasonable time after reaching majority age.

Failure to disaffirm within reasonable time after reaching majority acts as ratification (e.g., 1 year is too long in the absence of very special circumstances such as being out of the country)

How well did you know this?
1
Not at all
2
3
4
5
Perfectly
19
Q

Statute of Frauds - What must be in Writing?

  1. Sale of a golf cart for $750
  2. Sale of 1/20 acre of land for $300
  3. 3 year advertising services contract
  4. Sale of a mobile home by manufacturer
  5. Sale of a horse for $350
  6. Sale of a painting for $1,000
  7. Officer’s guarantee of a corporate note
  8. Sole proprietor’s guarantee of a business note
A
  1. Sale of a golf cart for $750 = YES
  2. Sale of 1/20 acre of land for $300 = YES
  3. 3 year advertising services contract = YES
  4. Sale of a mobile home by manufacturer = DEPENDS on price
  5. Sale of a horse for $350 = NO
  6. Sale of a painting for $1,000 = YES
  7. Officer’s guarantee of a corporate note = YES, guarantee of a another’s debt, the corportation is a “separate person”
  8. Sole proprietor’s guarantee of a business note = NO, this is original debt to the sole proprietor, not another’s debt.
How well did you know this?
1
Not at all
2
3
4
5
Perfectly
20
Q

The Parol Evidence Rule…

A

Applies to complete and unambiguous written contracts and makes any evidence that would modify or alter the written contract terms inadmissible.

This rule applies to any oral agreements made prior to or contemporaneous with the written contract.

The parol evidence rule will not allow evidence of prior agreements to be admitted as evidence. If a contract is established as a final expression of an agreement or a “total integration,” it is assumed that anything not in the final, written contract was not intended to be a part of the agreement.

An exception to the parol evidence rule allows evidence of “subsequent agreements” to be admitted into evidence.

How well did you know this?
1
Not at all
2
3
4
5
Perfectly
21
Q

Rice contracted with Locke to build an oil refinery for Locke. The contract provided that Rice was to use United pipe fittings. Rice did not do so. United learned of the contract and, anticipating the order, manufactured additional fittings. United sued Locke and Rice. United is

  • Entitled to recover from Rice only because Rice breached the contract.
  • Entitled to recover from either Locke or Rice because it detrimentally relied on the contract.
  • Not entitled to recover because it is a donee beneficiary.
  • Not entitled to recover because it is an incidental beneficiary.
A

Not entitled to recover because it is an incidental beneficiary. United is an incidental beneficiary because the contract was not made for its primary benefit. It will benefit from the contract if it is performed, but the parties did not have United’s benefit in mind when making the contract. Incidental beneficiaries may not sue to enforce contracts.

How well did you know this?
1
Not at all
2
3
4
5
Perfectly
22
Q

Union Bank lent $200,000 to Wagner. Union required Wagner to obtain a life insurance policy naming Union as beneficiary. While the loan was outstanding, Wagner stopped paying the premiums on the policy. Union paid the premiums, adding the amounts paid to Wagner’s loan. Wagner died and the insurance company refused to pay the policy proceeds to Union. Union may

  • Recover the policy proceeds because it is a creditor beneficiary.
  • Recover the policy proceeds because it is a donee beneficiary.
  • Not recover the policy proceeds because it is not in privity of contract with the insurance company.
  • Not recover the policy proceeds because it is only an incidental beneficiary.
A

Recover the policy proceeds because it is a creditor beneficiary. A person is a creditor beneficiary if two things are in place: one party to a contract in question owed the creditor money, and the contract in question was made specifically to satisfy that debt. Here, Wagner owed Union money and named Union as beneficiary in the life insurance contract to partially satisfy that debt. A creditor beneficiary may sue to enforce the contract.

How well did you know this?
1
Not at all
2
3
4
5
Perfectly
23
Q

West, Inc., and Barton entered into a contract. After receiving valuable consideration from Egan, West assigned its rights under the Barton contract to Egan. In which of the following circumstances would West not be liable to Egan?

  • West released Barton.
  • West breached the contract.
  • Egan released Barton.
  • Barton paid West.
A

Egan released Barton. Egan has all the rights of West based on the assignment. Thus, Egan can release Barton, discharging the Barton contract, and West has no further liability to Egan.

How well did you know this?
1
Not at all
2
3
4
5
Perfectly
24
Q

Pierce owed Duke $3,000. Pierce contracted with Lodge to paint Lodge’s house and Lodge agreed to pay Duke $3,000 to satisfy Pierce’s debt. Pierce painted Lodge’s house, but Lodge did not pay Duke the $3,000. In a lawsuit by Duke against Pierce and Lodge, who will be liable to Duke?

  • Pierce only.
  • Lodge only.
  • Both Pierce and Lodge.
  • Neither Pierce nor Lodge.
A

Both Pierce and Lodge. Pierce has made an assignment of his benefits under the contract or named Duke as a creditor beneficiary. Either way, Duke has contract rights against both of them as an assignee or a creditor beneficiary.

How well did you know this?
1
Not at all
2
3
4
5
Perfectly
25
Q

Which of the following statements is correct regarding the effect of the expiration of the period of the statute of limitations on a contract?

  • Once the period of the statute of limitations has expired, the contract is void.
  • The expiration of the period of the statute of limitations extinguishes the contract’s underlying obligation.
  • A cause of action barred by the statute of limitations may not be revived.
  • The running of the statute of limitations bars access to judicial remedies.
A

The running of the statute of limitations bars access to judicial remedies. The statute of limitations is a period of time within which a plaintiff must file an action in an appropriate court to receive judicial remedies. The period of time limitation usually begins at the time the cause for action occurs. Failure to file within the time period removes from the court the ability to grant a remedy. In a contract for the sale of goods, the statute of limitations period is four years but by agreement can be reduced to one year. A, B, and C are incorrect because although the plaintiff cannot seek a judicial remedy, it is a valid contract that can still be voluntarily completed, and for debts (an underlying obligation, for example), can be revived.

How well did you know this?
1
Not at all
2
3
4
5
Perfectly
26
Q

Which of the following actions could result in the discharge of a party to a contract?

  • Prevention of Performance?
  • Accord and satisfaction?
A

Both. An accord is acceptance of an offer and satisfaction by accord is a discharge of the contract. If an accord (for example, a partial payment check marked “paid in full” for an unliquidated debt) is accepted, this is a satisfaction and discharges the contract. If performance of a contract is prevented because of objective impossibility or commercial impracticability, performance is discharged.

How well did you know this?
1
Not at all
2
3
4
5
Perfectly
27
Q

To cancel a contract and to restore the parties to their original positions before the contract, the parties should execute a

  • Novation
  • Release
  • Rescission
  • Revocation
A

Rescission. A rescission is the undoing of a contract. Both sides are returned to their original positions, and the contractual obligations on both sides are discharged.

A novation takes place when one person takes over another’s contractual obligations, and the other is discharged. For example, a father might call a bank and say, “If you let my son out of his loan payments, I will make them.”

How well did you know this?
1
Not at all
2
3
4
5
Perfectly
28
Q

Under the UCC Sales Article, which of the following conditions will prevent the formation of an enforceable sale of goods contract?

  • Open price.
  • Open delivery.
  • Open quantity.
  • Open acceptance.
A

Open acceptance. Under the UCC, many terms may be left “open,” which means they will be determined at a later time. In this way, the UCC encourages the formation of agreements. Price, time of delivery and payment, and quantity may be left open. However, an acceptance must be made before a contract is enforceable. One can accept open terms, but the acceptance itself must be made. Before acceptance, the sides are merely carrying out unenforceable “preliminary negotiations.”

How well did you know this?
1
Not at all
2
3
4
5
Perfectly
29
Q

Under the Sales Article of the UCC, when a written offer has been made without specifying a means of acceptance but providing that the offer will only remain open for ten days, which of the following statements represent(s) a valid acceptance of the offer?

  • I. An acceptance sent by regular mail the day before the ten-day period expires that reaches the offeror on the eleventh day.
  • II. An acceptance faxed the day before the ten-day period expires that reaches the offeror on the eleventh day, due to a malfunction of the offeror’s printer.
A

Both I and II. The first scenario constitutes acceptance because of the mailbox rule. The second scenario constitutes acceptance because acceptances by mail or fax are deemed valid as soon as mailed or sent. As soon as the fax is properly “sent,” the acceptance becomes effective.

How well did you know this?
1
Not at all
2
3
4
5
Perfectly
30
Q

Under the Sales Article of the UCC, when a contract for the sale of goods stipulates that the seller ship the goods by common carrier “F.O.B. purchaser’s loading dock,” which of the parties bears the risk of loss during shipment?

  • The purchaser, because risk of loss passes when the goods are delivered to the carrier.
  • The purchaser, because risk of loss passes with the title.
  • The seller, because risk of loss passes only when the goods reach the purchaser’s loading dock.
  • The seller, because risk of loss remains with the seller until the goods are accepted by the purchaser.
A

The seller, because risk of loss passes only when the goods reach the purchaser’s loading dock. When the terms of a sale’s contract calls for delivery by a carrier at F.O.B. purchaser’s loading dock, “risk of loss” (in absence of express contract) passes from the seller to the buyer upon tender or delivery of the goods at the purchaser’s loading dock. Thus, the seller has the risk during shipment.

How well did you know this?
1
Not at all
2
3
4
5
Perfectly
31
Q

Under the Sales Article of the UCC, which of the following events will result in the risk of loss passing from a merchant seller to a buyer?

  • Tender of the goods at the seller’s place of business
  • Use of the seller’s truck to deliver the goods
A

NEITHER. The risk of loss starts with the seller. In neither of these situations does it pass to the buyer. The seller’s use of his or her own truck does nothing to affect the risk of loss until there is tender at buyer’s place of business or residence. Likewise, the seller’s placing of the goods at seller’s own place of business does not pass risk of loss if the buyer is to pick up the goods. Since the seller is a merchant, risk of loss does not pass until the buyer takes possession of the goods.

How well did you know this?
1
Not at all
2
3
4
5
Perfectly
32
Q

On September 10, Bell Corp. entered into a contract to purchase 50 lamps from Glow Manufacturing to be used in Bell Corp’s executive company offices. Bell prepaid 40% of the purchase price. Glow became insolvent on September 19 before segregating, in its inventory, the lamps to be delivered to Bell. Bell will not be able to recover the lamps because

  • Bell is regarded as a merchant.
  • The lamps were not identified to the contract.
  • Glow became insolvent fewer than 10 days after receipt of Bell’s prepayment.
  • Bell did not pay the full price at the time of purchase.
A

The lamps were not identified to the contract. The seller is the one who must identify goods by segregating them from general inventory and associating them with a specific contract before title would pass to the buyer. Since this has not yet been done, the buyer will have no rights in the goods under the contract.

How well did you know this?
1
Not at all
2
3
4
5
Perfectly
33
Q

Under the Sales Article of the UCC, which of the following statements is correct regarding a seller’s obligation under a F.O.B. destination contract?

  • The seller is required to arrange for the buyer to pick up the conforming goods at a specified destination.
  • The seller is required to tender delivery of conforming goods at a specified destination.
  • The seller is required to tender delivery of conforming goods at the buyer’s place of business.
  • The seller is required to tender delivery of conforming goods to a carrier who delivers to a destination specified by the buyer.
A

The seller is required to tender delivery of conforming goods at a specified destination. If the shipment terms require the seller to deliver goods under an F.O.B. destination contract, the seller is required to properly “tender” the goods to the buyer at the specific destination stated in the contract (not a destination specified by the buyer). This place can be other than the buyer’s place of business.

How well did you know this?
1
Not at all
2
3
4
5
Perfectly
34
Q

West purchased a painting from Noll, who is not in the business of selling art. West is picking up the painting from Noll. Noll tendered delivery of the painting after receiving payment in full from West. West informed Noll that West would be unable to take possession of the painting until later that day. Thieves stole the painting before West returned.

The risk of loss…

  • Remained with Noll, because West had not yet received the painting.
  • Remained with Noll, because the parties agreed on a later time of delivery.
  • Passed to West at the time the contract was formed and payment was made.
  • Passed to West on Noll’s tender of delivery.
A

Passed to West on Noll’s tender of delivery. In this case, there is a contract for delivery of goods without physical movement not represented by a document of title. Since Noll is a nonmerchant, risk of loss passes to West upon Noll’s tender of delivery. Thus, delivery need not be actually made for risk of loss to pass from seller to buyer. Delivery is tendered when, as was the case in this question, the goods are made available for a reasonable time for pick-up by the buyer.

How well did you know this?
1
Not at all
2
3
4
5
Perfectly
35
Q

Bond purchased a painting from Wool, who is not in the business of selling art. Wool tendered delivery of the painting after receiving payment in full from Bond. Bond informed Wool that Bond would be unable to take possession of the painting until later that day. Thieves stole the painting before Bond returned.

The risk of loss…

  • Passed to Bond at Wool’s tender of delivery.
  • Passed to Bond at the time the contract was formed and payment was made.
  • Remained with Wool, because the parties agreed on a later time of delivery.
  • Remained with Wool, because Bond had not yet received the painting.
A

Passed to Bond at Wool’s tender of delivery. In absence of agreement, where the buyer takes possession of the goods without a contractual obligation to ship or deliver by the seller, and the goods are not represented by a document of title, if the seller is a nonmerchant, risk of loss passes from seller to buyer at the time the seller tenders delivery. Bond suffers the loss.

How well did you know this?
1
Not at all
2
3
4
5
Perfectly
36
Q

Under the Sales Article of the UCC, most goods sold by merchants are covered by certain warranties.

An example of an express warranty would be a warranty of

  • Usage of trade.
  • Fitness for a particular purpose.
  • Merchantability.
  • Conformity of goods to sample.
A

Conformity of goods to sample.

There are three types of express warranty under the Sales Article of the UCC (if made as part of the bargain or sale), they are:

  1. Affirmations of fact or promises
  2. Description of the goods
  3. Sample or model - Bulk will conform exactly to the sample.

The UCC gives the buyer (unless disclaimed by the seller) certain implied warranties. Three of those implied warranties are the implied warranty of fitness for a particular purpose, the implied warranty of merchantability, and the implied warranty of the usage of trade. A sale by use of a sample is an express warranty the goods received conform to the sample.

How well did you know this?
1
Not at all
2
3
4
5
Perfectly
37
Q

Under the Sales Article of the UCC, which of the following circumstances best describes how the implied warranty of fitness for a particular purpose arises in a sale of goods transaction?

  • The buyer is purchasing the goods for a particular purpose and is relying on the seller’s skill or judgment to select suitable goods.
  • The buyer is purchasing the goods for a particular purpose and the seller is a merchant in such goods.
  • The seller knows the particular purpose for which the buyer will use the goods and knows the buyer is relying on the seller’s skill or judgment to select suitable goods.
  • The seller knows the particular purpose for which the buyer will use the goods and the seller is a merchant in such goods.
A

The seller knows the particular purpose for which the buyer will use the goods and knows the buyer is relying on the seller’s skill or judgment to select suitable goods.

It is the awareness of the seller that is the key.

How well did you know this?
1
Not at all
2
3
4
5
Perfectly
38
Q

Under the UCC Sales Article, an action for breach of the implied warranty of merchantability by a party who sustains personal injuries may be successful against the seller of the product only when

  • The seller is a merchant of the product involved.
  • An action based on negligence can also be successfully maintained.
  • The injured party is in privity of contract with the seller.
  • An action based on strict liability in tort can also be successfully maintained.
A

The seller is a merchant of the product involved.

To win a case under a breach of the implied warranty of merchantability, the plaintiff must show that a merchant sold the goods and that a breach of this warranty was the cause of an injury suffered.

How well did you know this?
1
Not at all
2
3
4
5
Perfectly
39
Q

Gray Fabricating Co. and Pine Corp. agreed orally that Pine would custom manufacture a processor for Gray at a price of $80,000.

After Pine completed the work at a cost of $60,000, Gray notified Pine that the processor was no longer needed. Pine is holding the processor and has requested payment from Gray. Pine has been unable to resell the processor for any price. Pine incurred storage fees of $1,000.

If Gray refuses to pay Pine and Pine sues Gray, the most Pine will be entitled to recover is?

  • $60,000
  • $61,000
  • $80,000
  • $81,000
A

$81,000

The point of contractual damages is to give the wronged party the “benefit of the bargain.” In this case, Gray expected $80,000 as that benefit in exchange for the processor. It did not get any of that expectation, and incurred an extra $1000 in expenses. To make it realize its expected benefit of $80,000, Gray must give Pine $81,000, as it is currently out $1000 after paying the storage. It will get $81,000.

How well did you know this?
1
Not at all
2
3
4
5
Perfectly
40
Q

On May 6, Maple entered into a signed contract with Ard, whereby Maple was to sell Ard a painting having a fair market value of $350,000 for $130,000. Maple believed the painting was worth only $130,000. Unknown to either party the painting had been destroyed by fire on May 4. If Ard sued Maple for breach of contract, Maple’s best defense is

  • Risk of loss had passed to Ard.
  • Lack of adequate consideration.
  • Mutual mistake.
  • Unconscionability.
A

A mutual mistake occurs when two parties intentionally enter into an agreement, but under an erroneous conviction. In this case, at the time of entering the contract, both parties reasonably believed that the destroyed painting was still in existence. Thus Maple’s best defense is that there was a mutual mistake of an existing fact, which renders the contract voidable.

Mutual mistake (i.e., by both parties) about existence, identity, or important characteristics of subject matter in contract makes contract voidable by either party

How well did you know this?
1
Not at all
2
3
4
5
Perfectly
41
Q

Assuming all other requirements have been met, which of the following terms generally must be included in a writing in order to satisfy the UCC Statute of Frauds regarding the sale of goods for?

  • Price
  • Quantity
  • Time of payment
A
  • Price = NO
  • Quantity = YES
  • Time of payment = NO

In order to satisfy the UCC Statute of Frauds regarding the sale of goods, only the quantity term must be included in the writing. Failure to include the time of payment will not cause the writing to fail to satisfy the UCC Statute of Frauds because a reasonable time of payment will be inferred. Failure to include other terms of the contract, including price, will not result in an insufficient writing either. The Code implies that the parties will in “good faith” determine a reasonable price.

How well did you know this?
1
Not at all
2
3
4
5
Perfectly
42
Q

Which of the following will be legally binding on all parties despite lack of consideration?

  1. An irrevocable oral promise by a merchant to keep an offer open for 60 days.
  2. A promise to donate money to a charity which the charity relied upon in incurring large expenditures.
  3. A promise to pay for the college education of the child of a person who saved the promisor’s life.
  4. A signed modification to a contract to purchase a parcel of land.
A

A promise to donate money to a charity which the charity relied upon in incurring large expenditures.

A promise to donate money to a charity which the charity relied upon in incurring large expenditures is a situation involving promissory estoppel. Promissory estoppel acts as a substitute for consideration and renders the promise enforceable. The elements necessary for promissory estoppel are:

  1. detrimental reliance on a promise,
  2. reliance on the promise is reasonable and foreseeable, and
  3. damage results (injustice) if the promise is not enforced.
How well did you know this?
1
Not at all
2
3
4
5
Perfectly
43
Q

Ward is attempting to introduce oral evidence in an action relating to a written contract between Ward and Weaver. Weaver has pleaded the parol evidence rule. Ward will be prohibited from introducing parol evidence if it relates to

  1. A modification made several days after the contract was executed.
  2. A change in the meaning of an unambiguous provision in the contract.
  3. Fraud in the inducement.
  4. An obvious error in drafting.
A

A change in the meaning of an unambiguous provision in the contract. The parol evidence rule prohibits the presentation as evidence of any prior or contemporaneous oral statements for the purpose of modifying or changing a written agreement intended by the parties to be the final and complete expression of their contract. It would bar the admission of evidence which relates to a change in an unambiguous provision in the contract but would not bar the admission of evidence which clarifies an ambiguous provision.

How well did you know this?
1
Not at all
2
3
4
5
Perfectly
44
Q

Montrose sent Bilbo a written offer to sell his tract of land located in Majorsville for $50,000. The parties were engaged in a separate dispute. The offer stated that it would be irrevocable for 30 days if Bilbo would promise to refrain from suing Montrose during this time. Bilbo promptly delivered a promise not to sue during the term of the offer and to forego suit if she accepted the offer. Montrose subsequently decided that the possible suit by Bilbo was groundless and therefore phoned Bilbo and revoked the offer 10 days after making it. Bilbo mailed an acceptance on the 30th day. Montrose did not reply. Under the circumstances

  1. Montrose’s offer was supported by consideration, and was irrevocable for the 30 day period.
  2. Bilbo’s promise was accepted by Montrose by his silence.
  3. Montrose’s revocation, not being in writing, was invalid.
  4. Montrose’s written offer would be irrevocable even without consideration.
A

Montrose’s offer was supported by consideration, and was irrevocable for the 30 day period. Bilbo’s promise to forego suit would be sufficient consideration to create an option offer. An option offer is irrevocable for the stated time period.

How well did you know this?
1
Not at all
2
3
4
5
Perfectly
45
Q

Walker tells Side that she will hire him for 10 months’ work for $6,000 per month starting 6 months from now. Side accepts. When Side proceeds to work for Walker, Walker refuses, pointing out that their contract was not in writing. Side admits the lack of a written contract but claims they still have an enforced contract. Who wins?

  1. Side because Walker admitted to having an oral contract with Side.
  2. Side because the actual employment period is for 6 months, that is, less than 1 year.
  3. Walker because this contract needed to be in writing.
  4. Walker because this contract was for $6,000 and thus needed to be in writing.
A

Walker because this contract needed to be in writing. This contract could not be completed until 16 months after the making of the contract. Therefore, it had to be written since it fell under the Statute of Frauds since it could not be completed within 1 year of the making of the contract.

46
Q

Smith contracted to perform for $500 certain services for Jones. Jones claimed that the services had been performed poorly. Because of this, Jones sent Smith a check for only $425. Marked clearly on the check was “payment in full.” Smith crossed out the words “payment in full” and cashed the check. Assuming that there was a bona fide dispute as to whether Smith had in fact performed the services poorly, the majority of courts would hold that

  1. The debt is liquidated, and Smith can collect the remaining $75.
  2. The debt is liquidated, but Jones by adding the words “payment in full” cancelled the balance of the debt owed.
  3. The debt is unliquidated and the cashing of the check by Smith completely discharged the debt.
  4. The debt is unliquidated, but the crossing out of the words “payment in full” by Smith revives the balance of $75 owed.
A

The debt is unliquidated and the cashing of the check by Smith completely discharged the debt.

At the time the contract was made, the debt was liquidated since the amount was certain ($500). However, the bona fide dispute changed the debt to an unliquidated debt. Payment of a lesser sum to discharge an unliquidated debt will be effective if accepted as payment in full since each party gives consideration in the form of forfeiting a claim to dispute the amount of the debt. Smith’s cashing of the check was acceptance of a settlement for the full amount of the debt.

47
Q

On reaching majority, a minor may ratify a contract in any of the following ways except by

  1. Failing to disaffirm within a reasonable time after reaching majority.
  2. Orally ratifying the entire contract.
  3. Acting in a manner that amounts to ratification.
  4. Affirming, in writing, some of the terms of the contract.
A

Affirming, in writing, some of the terms of the contract.

The entire contract must be ratified in order for ratification to be effective. This rule holds true regardless of whether a partial affirmation is in writing.

Minor may choose to ratify within a reasonable time after reaching age of majority:

  • By words, either orally or in writing but must ratify all, or
  • By actions that indicate ratification
  • Ratification prior to majority is not effective
48
Q

On April 2, Jet Co. wrote to Ard, offering to buy Ard’s building for $350,000. The offer contained all of the essential terms to form a binding contract and was duly signed by Jet’s president. It further provided that the offer would remain open until May 30 and an acceptance would not be effective until received by Jet. On April 10, Ard accepted Jet’s offer by mail. The acceptance was received by Jet on April 14. Assume that on April 11 Jet sent a telegram to Ard revoking its offer and that Ard received the telegram on April 12. Under the circumstances,

  1. A contract was formed on April 10.
  2. A contract was formed on April 14.
  3. Jet’s revocation effectively terminated its offer on April 12.
  4. Jet’s revocation effectively terminated its offer on April 11.
A

Jet’s revocation effectively terminated its offer on April 12.

An acceptance is considered to be effective when sent by the offeree as long as the acceptance was sent by a means specified in the offer or by the same means used to communicate the offer. However, the offeror may stipulate that acceptance is effective only when received by the offeror. Therefore, although Ard sent an acceptance on April 10, it would not become effective until the date it was received by Jet, April 14. In this situation, Jet’s revocation was received by Ard prior to the time that Ard’s acceptance reached Jet. Since a revocation is effective when received, Jet’s revocation effectively terminated its offer on April 12.

49
Q

Generally, which one of the following transfers will be valid without the consent of the other parties?

  1. The assignment by the lessee of a lease contract where rent is a percentage of sales.
  2. The assignment by a purchaser of goods of the right to buy on credit without giving security.
  3. The assignment by an architect of a contract to design a building.
  4. The assignment by a patent holder of the right to receive royalties.
A

The assignment by a patent holder of the right to receive royalties.

In general, a party’s rights in a contract are assignable without the consent of the other parties. However, the following are situations in which this general rule does not apply and consent of the other parties would be required for a valid transfer to occur:

  1. the contract involves personal services, credit, trust or confidence,
  2. a provision of the contract or statute prohibits assignment, and
  3. the assignment would materially change the risk or obligations of the other party. Since the assignment by a patent holder of the right to receive royalties would not alter the rights of the other parties to the contract, a valid transfer could be made without the consent of these parties.
50
Q

Macy agreed orally to repaint Rich’s home for $2,000. Rich required that it be done in 16 months although Macy said he might have it done in 10 months. The work actually took 14 months. Does this contract fall within the Statute of Frauds?

  1. Yes, because the contract was for $2,000.
  2. Yes, because the work actually took 14 months to complete.
  3. Yes, because Rich required that the contract be completed within 16 months.
  4. No, because the work could have been completed within 1 year.
A

NO.

Since this contract could have been completed within 1 year, it does not fall within the Statute of Frauds and therefore can be oral.

51
Q

Omega Corp. owned a factory that was encumbered by a mortgage securing Omega’s note to Eagle Bank. Omega sold the factory to Spear, Inc., which assumed the mortgage note. Later, Spear defaulted on the note, which had an outstanding balance of $15,000. To recover the outstanding balance, Eagle

  1. May sue Spear only after suing Omega.
  2. May sue either Spear or Omega.
  3. Must sue both Spear and Omega.
  4. Must sue Spear first and then proceed against Omega for any deficiency.
A

May sue either Spear or Omega. When a purchaser of property assumes a mortgage note, s/he promises the mortgagor that s/he will pay the mortgage debt. However, the mortgagor is not relieved of his/her own duty to pay the debt, but the mortgagor does have a right to insist that the purchaser make payment. The mortgagee can look to either party for payment–the mortgagor on the basis of the original mortgage agreement, or the purchaser on the basis of being a third-party beneficiary of the assumption agreement. Thus, in this case, Eagle can sue either Spear or Omega.

52
Q

For which of the following contracts will a court generally grant the remedy of specific performance?

  1. A contract for the sale of a patent.
  2. A contract of employment.
  3. A contract for the sale of fungible goods.
  4. A contract for the sale of stock that is traded on a national stock exchange.
A

A contract for the sale of a patent.

Specific performance is generally used when money damages will not suffice such as when the subject matter of the contract is unique or rare. A patent sale typically satisfies this rule.

Specific performance—compels performance promised

  1. Used when money damages will not suffice (e.g., when subject matter is unique, or rare, as in contract for sale of land)
  2. Injured party may seek compensatory damages if s/he reasonably chooses them
  3. Not available to compel personal services
53
Q

Lark, CPA, entered into a signed contract with Bale Corp. to perform management advisory services for Bale. If Lark repudiates the contract prior to the date performance is due to begin, which of the following is not correct?

  1. Bale could successfully maintain an action for breach of contract prior to the date performance is due to begin.
  2. Bale can obtain a judgment for the monetary damages it incurred as a result of the repudiation.
  3. Bale could successfully maintain an action for breach of contract after the date performance was due to begin.
  4. Bale can obtain a judgment ordering Lark to perform.
A

Bale can obtain a judgment ordering Lark to perform.

The contract entered into by Lark is a personal service contract. The remedy of specific performance is not available for breach of a personal service contract. It would violate the constitutional amendment prohibiting involuntary servitude. Therefore, Bale will be unable to obtain a judgment ordering Lark to perform.

54
Q

On November 1, Yost sent a telegram to Zen offering to sell a rare vase. The offer required that Zen’s acceptance telegram be sent on or before 5:00 P.M. on November 2. On November 2, at 3:00 P.M., Zen sent an acceptance by overnight mail. It did not reach Yost until November 5. Yost refused to complete the sale to Zen. Is there an enforceable contract?

  1. Yes, because the acceptance was made within the time specified.
  2. Yes, because the acceptance was effective when sent.
  3. No, because Zen did not accept by telegram.
  4. No, because the offer required receipt of the acceptance within the time specified.
A

No, because Zen did not accept by telegram.

Normally, an acceptance is considered to be effective when sent by the offeree under the UCC as long as the acceptance was sent by a reasonable method. This rule can be changed if so stated in the offer. In this situation, Zen did not accept by the means specified in the offer and, therefore, the acceptance was not effective when sent and a valid contract was not formed.

55
Q

On May 25, Smith contracted with Jackson to repair Smith’s cabin cruiser. The work was to begin on May 31. On May 26, the boat, while docked at Smith’s pier, was destroyed by arson. Which of the following statements is correct with regard to the contract?

  1. Smith would not be liable to Jackson because of mutual mistake.
  2. Smith would be liable to Jackson for the profit Jackson would have made under the contract.
  3. Jackson would not be liable to Smith because performance by the parties would be impossible.
  4. Jackson would be liable to repair another boat owned by Smith.
A

Jackson would not be liable to Smith because performance by the parties would be impossible.

When performance of a contract becomes objectively impossible, both parties to the contract are discharged from their contractual obligations. The destruction of Smith’s cabin cruiser through no fault of either party constitutes objective impossibility of performance. Therefore, both parties are relieved of their contractual obligations.

Note: But mere fact of performance becoming more costly does not excuse performance
EXAMPLE: A agreed to sell a specified quantity of corn to B at specified prices. He had planned to sell his own corn until his crop was destroyed. Even though he may make less profit or even suffer a loss, he can still fulfill the contract by purchasing the corn from others to resell to B under his contract.

56
Q

Walker tells Side that she will hire him for 10 months’ work for $6,000 per month starting 6 months from now. Side accepts. When Side proceeds to work for Walker, Walker refuses, pointing out that their contract was not in writing. Side admits the lack of a written contract but claims they still have an enforced contract. Who wins?

  1. Side because Walker admitted to having an oral contract with Side.
  2. Side because the actual employment period is for 6 months, that is, less than 1 year.
  3. Walker because this contract needed to be in writing.
  4. Walker because this contract was for $6,000 and thus needed to be in writing.
A

Walker because this contract needed to be in writing.

This contract could not be completed until 16 months after the making of the contract. Therefore, it had to be written since it fell under the Statute of Frauds since it could not be completed within 1 year of the making of the contract.

57
Q

Mayker, Inc. and Oylco contracted to have Oylco be the exclusive provider of Mayker’s fuel oil for 3 months. The stated price was subject to increases of up to a total of 10% if the market price increased. The market price rose 25% and Mayker tripled its normal order. Oylco seeks to avoid performance. Oylco’s best argument in support of its position is that

  1. There was no meeting of the minds.
  2. The contract was unconscionable.
  3. The quantity was not definite and certain enough.
  4. Mayker ordered amounts of oil unreasonably greater than its normal requirements.
A

Mayker ordered amounts of oil unreasonably greater than its normal requirements.

The agreement between Mayker, Inc. and Oylco is a requirements contract. Under a requirements contract, the seller cannot be required to sell amounts unreasonably disproportionate to normal requirements of the buyer. Thus, Mayker’s ordering of unreasonably disproportionate amounts of oil would be a breach of the contract and would allow Oylco to avoid performance.

58
Q

Price and White entered into an all-inclusive written contract involving the purchase of a building. Their written agreement contained provisions concerning renovation work to the building to be completed by Price. This aspect of the written contract was modified by a contemporaneous oral agreement between the parties. Price relies upon the parol evidence rule to support his position that the written contract is binding on the parties. Which of the following is correct?

  1. Since the Statute of Frauds was satisfied in respect to the contract for the purchase of the building, the parol evidence does not apply.
  2. Since the oral agreement related to the same subject matter as the written contract, the parol evidence rule does not apply.
  3. White will be precluded from introducing into evidence proof of the oral agreement because of the parol evidence rule.
  4. The parol evidence rule does not apply to contemporaneous oral agreements.
A

White will be precluded from introducing into evidence proof of the oral agreement because of the parol evidence rule.

This answer is correct because when there is a written agreement intended by the parties to be a fully integrated and completed contract, the parties are barred by the parol evidence rule from introducing prior or contemporaneous evidence which contradicts the terms of the agreement. Thus, White will be precluded from introducing into evidence proof of the oral agreement since it occurred contemporaneously with the written agreement.

59
Q

On April 1, 2015, Crowe and Greene formed Apex Corporation. The same day Crowe paid $150,000 for 500 shares of Apex common stock, and Greene transferred land and building to Apex in exchange for 500 shares of common stock. The land and building had an adjusted basis to Greene of $120,000, a fair market value of $200,000, and was subject to a mortgage of $60,000 on April 1, 2015. The mortgage was assumed by Apex. Apex had no other shares of stock outstanding on April 1, 2015. The basis of the land and building to Apex on April 1, 2015, is

  1. $ 60,000
  2. $120,000
  3. $140,000
  4. $150,000
A

$120,000

The land and building were transferred by Greene in a nontaxable Sec. 351 transfer to a controlled corporation. The basis of the land and building to Apex would be the same as Greene’s adjusted basis, increased by any gain recognized by Greene. Since Greene did not receive any boot, no gain was recognized by him. Apex’s basis for the land and building is $120,000.

60
Q

ABC has 200 shares of voting common stock outstanding. XYZ has decided to acquire 90 percent of the ABC stock solely in exchange for 50 percent of its voting stock. ABC will become XYZ’s subsidiary after the transaction. Which of the following statements is true?

  1. XYZ must acquire 100 percent of ABC for the transaction to qualify as a reorganization.
  2. The transaction is a reorganization.
  3. XYZ must issue at least 60 percent of its stock for the transaction to qualify as a reorganization.
  4. ABC must surrender assets to XYZ to qualify as a reorganization.
A

The transaction is a reorganization.

The 80% control test is met since 90% of ABC’s voting stock is being acquired. XYZ is using its voting stock as required for B reorganizations.

61
Q

Pursuant to a plan of corporate reorganization adopted in July 2015, Gow exchanged 500 shares of Lad Corp. common stock that he had bought in January 2015 at a cost of $5,000 for 100 shares of Rook Corp. common stock having a fair market value of $6,000.

Gow’s recognized gain on this exchange was

  1. $1,000 long-term capital gain.
  2. $1,000 short-term capital gain.
  3. $1,000 ordinary income.
  4. $0.
A

$0.

If taxpayer receives stocks or securities under a plan of reorganization from a corporation included in the reorganization, the taxpayer does not recognize a gain or loss from the transaction. However, if the taxpayer receives boot, the transaction is taxable up to the amount of the boot.

Since Gow received the Rook Corp. stock solely in exchange for his Lad Corp. stock under a plan of reorganization and did not receive any boot, the transaction would be tax-free for Gow.

62
Q

Corporations A and B combine in a qualifying reorganization, and form Corporation C, the only surviving corporation.

This reorganization is tax-free to the

  • Shareholders
  • Corporation
A

YES for BOTH.

Corporate reorganizations generally are tax-free for both shareholders and the corporation.

In this case, the reorganization would be viewed as a Type A: Merger or Consolidation, which qualifies for tax-free treatment for both shareholders and the corporation.

This response correctly indicates that the transaction would be tax-free for both shareholders and the corporation and, therefore, it is correct.

63
Q

P corporation acquired the assets of its wholly-owned subsidiary, S corporation, under a plan that qualified as a tax-free complete liquidation of S. Which of the following of S’s unused carryovers may be transferred to P?

  1. Excess charitable contributions.
  2. Net operating loss.
  3. Both of the above are transferred.
  4. None of the above are transferred.
A

Both of the above are transferred.

Tax attributes of the subsidiary transfer to the parent after a tax-free liquidation of the subsidiary into the parent.

64
Q

Love granted Nelson a written option to buy a tract of land in an industrial park. The option stated that it was irrevocable for 11 days and was given for $20 and other valuable consideration. The $20 was not paid and there was no other valuable consideration. Which of the following is a correct statement regarding the option in question?

  1. Since real property is involved, Nelson’s acceptance must be contained in a signed writing if Nelson is to enforce it against Love.
  2. It is an option contract enforceable for the 11-day period.
  3. Acceptance must be received at Love’s place of business before expiration of the 11 days.
  4. It is unenforceable because it lacks consideration.
A

It is unenforceable because it lacks consideration.

The option stated that it was given for $20 and other valuable consideration. Neither the $20 nor the other valuable consideration was present; therefore, the option is unenforceable because it lacks consideration.

65
Q

Kent Construction Company contracted to construct four garages for Magnum, Inc., according to specifications provided by Magnum. Kent deliberately substituted 2 x 4s for the more expensive 2 x 6s called for in the contract in all places where the 2 x 4s would not be readily detected. Magnum’s inspection revealed the variance and Magnum is now withholding the final payment on the contract. The contract was for $100,000, and the final payment would be $25,000. Damages were estimated to be $15,000. In a lawsuit for the balance due, Kent will

  1. Prevail on the contract, less damages of $15,000, because it has substantially performed.
  2. Prevail because the damages in question were not substantial in relation to the contract amount.
  3. Lose because the law unqualifiedly requires literal performance of such contracts.
  4. Lose all rights under the contract because it has intentionally breached it.
A

Lose all rights under the contract because it has intentionally breached it.

Under the doctrine of substantial performance, a contract obligation may be discharged even though the performance tendered was not in complete conformity with the terms of the agreement. Under this doctrine, if it can be shown that the defect in performance was only minor in nature, a good-faith effort was made to conform completely with the terms of the agreement, and if the performing party is willing to accept a decrease in compensation equivalent to the amount of the minor defect in performance, the contractual obligation will be discharged. Since Kent did not make a good-faith effort to conform to the terms of the agreement, but in fact intentionally breached it, their obligation will not be discharged and they will lose all rights under the contract.

66
Q

Under the parol evidence rule, oral evidence will be excluded if it relates to

  1. A contemporaneous oral agreement relating to a term in the contract.
  2. Failure of a condition precedent.
  3. Lack of contractual capacity.
  4. A modification made several days after the contract was executed.
A

A contemporaneous oral agreement relating to a term in the contract.

The parol evidence rule prohibits the presentation as evidence of any prior or contemporaneous oral statements pertaining to a written agreement intended by the parties to be the final and complete expression of their contract. Two of the exceptions to this rule include the following: (1) to show invalidity of the contract due to a lack of contractual capacity or reality of consent, and (2) to show the existence or failure of a condition precedent.

67
Q

Water Works had a long-standing policy of offering employees $100 for suggestions actually used. Due to inflation and a decline in the level and quality of suggestions received, Water Works decided to increase the award to $500. Several suggestions were under consideration at that time. Two days prior to the public announcement of the increase to $500, a suggestion by Farber was accepted and put into use. Farber is seeking to collect $500. Farber is entitled to

  1. $500 because Water Works had decided to pay that amount.
  2. $500 because the suggestion submitted will be used during the period that Water Works indicated it would pay $500.
  3. $100 in accordance with the original offer.
  4. Nothing if Water Works chooses not to pay since the offer was gratuitous.
A

$100 in accordance with the original offer.

An offeree cannot accept an offer unless the offeree knows of the existence of the offer; otherwise there is no objective meeting of the minds. Farber does not know of the increase of the monetary award to $500, and submitted his suggestion as an acceptance to the terms of the offer of which he knew (i.e., the $100). Even though Water Works later raised the amount and will use the suggestion in the later period, Farber only has a right to receive $100.

68
Q

In year 1, Dart bought an office building from Graco under a written contract signed only by Dart. In year 27, Dart discovered that Graco made certain false representations during their negotiations concerning the building’s foundation. Dart could have reasonably discovered the foundation problems by year 7. Dart sued Graco claiming fraud in the formation of the contract. Which of the following statements is correct?

  1. The parol evidence rule will prevent the admission into evidence of proof concerning Dart’s allegations.
  2. Dart will be able to rescind the contract because both parties did not sign it.
  3. Dart must prove that the alleged misrepresentations were part of the written contract because the contract involved real estate.
  4. The statute of limitations would likely prevent Dart from prevailing because of the length of time that has passed.
A

The statute of limitations would likely prevent Dart from prevailing because of the length of time that has passed.

The statute of limitations requires the plaintiff to file a lawsuit within a specified period of time after the cause of action accrues (e.g., breach). A plaintiff who fails to file suit within the period of the statute of limitations loses the right to recover on the claim. The time limit related to contract disputes varies by state. Common provisions are 5 years after breach of an oral contract and 10 years after breach of a written contract. Clearly, the statute of limitations would likely bar Dart’s recovery, due to 35 years passing since the contract was formed.

69
Q

The intent, or scienter, element necessary to establish a cause of action for fraud will be met if the plaintiff can show that the

  1. Defendant made a misrepresentation with a reckless disregard for the truth.
  2. Defendant made a false representation of fact.
  3. Plaintiff actually relied on the defendant’s misrepresentation.
  4. Plaintiff justifiably relied on the defendant’s misrepresentation.
A

Defendant made a misrepresentation with a reckless disregard for the truth.

Scienter is defined as the intent to mislead. This element, necessary in establishing fraud, is met by showing that the defendant had knowledge of a misrepresentation with the intent to mislead, or showed a reckless disregard for the truth.

70
Q

Sand sold a warehouse he owned to Quick Corp. The warehouse was encumbered by an outstanding mortgage securing Sand’s note to Security Bank. Quick assumed Sand’s note and mortgage at the time it purchased the warehouse from Sand. Within 3 months, Quick defaulted on the note and Security Bank commenced a mortgage foreclosure action. The proceeds of the resulting foreclosure sale were less than the outstanding balance on the note.

As to the contract between Sand and Quick, Security is

  1. A third-party creditor beneficiary.
  2. A third-party donee beneficiary.
  3. A third-party incidental beneficiary.
  4. Not a beneficiary.
A

A third-party creditor beneficiary.

A third-party beneficiary contract is present when the contracting parties enter into an agreement intended to benefit a third party. Security Bank is a third-party creditor beneficiary because Quick is contracting with Sand, the debtor, to pay a debt owed to Security, the creditor.

71
Q

On May 25, Fresno sold Bronson, a minor, a used computer. On June 1, Bronson reached the age of majority. On June 10, Fresno wanted to rescind the sale. Fresno offered to return Bronson’s money and demanded that Bronson return the computer. Bronson refused, claiming that a binding contract existed. Bronson’s refusal is

  1. Not justified, because Fresno is not bound by the contract unless Bronson specifically ratifies the contract after reaching the age of majority.
  2. Not justified, because Fresno does not have to perform under the contract if Bronson has a right to disaffirm the contract.
  3. Justified, because Bronson and Fresno are bound by the contract as of the date Bronson reached the age of majority.
  4. Justified, because Fresno must perform under the contract regardless of Bronson’s minority.
A

Justified, because Fresno must perform under the contract regardless of Bronson’s minority.

The requirement is to determine whether a binding contract existed with respect to Fresno. Since Fresno was not a minor, s/he is required to perform under the contract unless Bronson exercises his or her right to disaffirm the contract in a timely fashion. Bronson did not do this.

72
Q

On December 1, Gem orally contracted with Mason for Mason to manage Gem’s restaurant for one year starting the following January 1. They agreed that Gem would pay Mason $40,000 and that Mason would be allowed to continue to work for Gem if “everything worked out.” On June 1, Mason quit to take a better paying job, alleging that the contract violated the Statute of Frauds. What will be the outcome of a suit by Gem for breach of contract?

  1. Gem will win because the contract was executory.
  2. Gem will win because the contract was for services, not goods.
  3. Gem will lose because the contract could not be performed within one year.
  4. Gem will lose because the contract required payment of more than $500.
A

Gem will lose because the contract could not be performed within one year.

The Statute of Frauds requires that contracts that cannot be performed within one year be evidenced by writing. The one year period begins from the date the contract is formed.

73
Q

Jones owned an insurance policy on her life, on which she paid all the premiums. Smith was named the beneficiary. Jones died and the insurance company refused to pay the insurance proceeds to Smith. An action by Smith against the insurance company for the insurance proceeds will be

  1. Successful because Smith is a third-party donee beneficiary.
  2. Successful because Smith is a proper assignee of Jones’ rights under the insurance policy.
  3. Unsuccessful because Smith was not the owner of the policy.
  4. Unsuccessful because Smith did not pay any of the premiums.
A

Successful because Smith is a third-party donee beneficiary.

The beneficiary of a life insurance contract is an intended third-party donee beneficiary, and as such, can maintain an action against the insurance company in the event of default. In this case Smith, as the third-party donee beneficiary, will be successful in an action against the insurance company.

74
Q

Which of the following types of mistake will generally make a contract unenforceable and allow it to be rescinded?

  1. A unilateral mistake of fact.
  2. A mutual mistake of fact.
  3. A unilateral mistake of value.
  4. A mutual mistake of value.
A

A mutual mistake of fact.

The requirement is to identify the mistake that will make a contract unenforceable. This answer is correct because a mutual mistake of fact typically causes a contract to be unenforceable and allows it to be rescinded.

75
Q

Kram sent Fargo, a real estate broker, a signed offer to sell a specified parcel of land to Fargo for $250,000. Kram, an engineer, had inherited the land. On the same day that Kram’s letter was received, Fargo telephoned Kram and accepted the offer. Which of the following statements is correct under the common-law statute of frauds?

  1. No contract could be formed because Fargo’s acceptance was oral.
  2. No contract could be formed because Kram’s letter was signed only by Kram.
  3. A contract was formed and would be enforceable against both Kram and Fargo.
  4. A contract was formed but would be enforceable only against Kram.
A

A contract was formed but would be enforceable only against Kram.

The requirement is to identify the correct application of the statute of fraud to a contract for land. The contract for the land was formed but it would only be enforceable against Kram because Kram’s offer was written and signed. Under the common-law statute of frauds, the contract is not enforceable against Fargo because s/he orally accepted the offer rather than agreeing with a signed writing.

76
Q

Myers entered into a contract to purchase a valuable rare coin from Eisen. Myers tendered payment which was refused by Eisen. Upon Eisen’s breach, Myers brought suit to obtain the coin. The court will grant Myers

  1. Compensatory damages.
  2. Specific performance.
  3. Reformation.
  4. Restitution.
A

Specific performance.

The court will grant specific performance of a contract as a remedy if money damages will not be adequate to compensate the nonbreaching party. Since the coin is rare (unique) and cannot be readily purchased in the market, the court will grant specific performance to Myers.

77
Q

Ward is attempting to introduce oral evidence in an action relating to a written contract between Ward and Weaver. Weaver has pleaded the parol evidence rule. Ward will be prohibited from introducing parol evidence if it relates to

  1. A modification made several days after the contract was executed.
  2. A change in the meaning of an unambiguous provision in the contract.
  3. Fraud in the inducement.
  4. An obvious error in drafting.
A

A change in the meaning of an unambiguous provision in the contract.

The parol evidence rule prohibits the presentation as evidence of any prior or contemporaneous oral statements for the purpose of modifying or changing a written agreement intended by the parties to be the final and complete expression of their contract. It would bar the admission of evidence which relates to a change in an unambiguous provision in the contract but would not bar the admission of evidence which clarifies an ambiguous provision.

78
Q

In September, Cobb Company contracted with Thrifty Oil Company for the delivery of 100,000 gallons of heating oil at the price of $.75 per gallon at regular specified intervals during the forthcoming winter. Due to an unseasonably warm winter, Cobb took delivery on only 70,000 gallons. In a suit against Cobb for breach of contract, Thrifty will

  1. Lose, because Cobb acted in good faith.
  2. Lose, because both parties are merchants and the UCC recognizes commercial impracticability.
  3. Win, because this is a requirements contract.
  4. Win, because the change of circumstances could have been contemplated by the parties.
A

Win, because the change of circumstances could have been contemplated by the parties.

The contract entered into by Cobb Company with Thrifty Oil Company was a valid contract for a specified quantity of heating oil. Therefore, Cobb is obligated to purchase the entire 100,000 gallons from Thrifty as agreed upon in the contract. The fact that there was an unseasonably warm winter will not change Cobb’s obligation to Thrifty since this was a change of circumstances which could be foreseen by the parties.

79
Q

Montrose sent Bilbo a written offer to sell his tract of land located in Majorsville for $50,000. The parties were engaged in a separate dispute. The offer stated that it would be irrevocable for 30 days if Bilbo would promise to refrain from suing Montrose during this time. Bilbo promptly delivered a promise not to sue during the term of the offer and to forego suit if she accepted the offer. Montrose subsequently decided that the possible suit by Bilbo was groundless and therefore phoned Bilbo and revoked the offer 10 days after making it. Bilbo mailed an acceptance on the 30th day. Montrose did not reply. Under the circumstances

  1. Montrose’s offer was supported by consideration, and was irrevocable for the 30 day period.
  2. Bilbo’s promise was accepted by Montrose by his silence.
  3. Montrose’s revocation, not being in writing, was invalid.
  4. Montrose’s written offer would be irrevocable even without consideration.
A

Montrose’s offer was supported by consideration, and was irrevocable for the 30 day period.

Bilbo’s promise to forego suit would be sufficient consideration to create an option offer. An option offer is irrevocable for the stated time period.

A promise to give up a legal right is valid consideration. An offer supported by consideration is an option.

80
Q

Bond and Spear orally agreed that Bond would buy a car from Spear for $475. Bond paid Spear a $100 deposit. The next day, Spear received an offer of $575, the car’s fair market value. Spear immediately notified Bond that Spear would not sell the car to Bond and returned Bond’s $100. If Bond sues Spear and Spear defends on the basis of the Statute of Frauds, Bond will probably

  1. Lose, because the agreement was for less than the fair market value of the car.
  2. Win, because the agreement was for less than $500.
  3. Lose, because the agreement was not in writing and signed by Spear.
  4. Win, because Bond paid a deposit.
A

Win, because the agreement was for less than $500.

The Statute of Frauds requires that a contract for the sale of goods for $500 or more be in writing in order to be enforceable. In this situation, since the agreed upon price was less than $500, the oral agreement is enforceable and Bond will probably win.

81
Q

Foster Co. and Rice executed a contract by which Foster was to sell a warehouse to Rice for $270,000. The contract required Rice to pay the entire $270,000 at the closing. Foster has refused to close the sale of the warehouse to Rice. If Rice commences a lawsuit against Foster, what relief would Rice likely be entitled to?

  1. Specific performance or compensatory damages.
  2. Specific performance and compensatory damages.
  3. Compensatory damages or punitive damages.
  4. Compensatory damages and punitive damages.
A

Specific performance or compensatory damages.

The remedy of specific performance is used when money damages will not sufficiently compensate the afflicted party due to the unique nature of the subject matter of the contract. In a contract for the sale of land, the buyer has the right to enforce the agreement by seeking the remedy of specific performance because real property is considered unique. Another remedy for this breach of contract would be for the buyer to seek compensatory damages. If the buyer desires, s/he may seek this remedy instead of specific performance. However, in this situation, Rice could only sue for either specific performance or compensatory damages but would not be entitled to both remedies.

82
Q

Paco Corp., a building contractor, offered to sell Preston several pieces of used construction equipment. Preston was engaged in the business of buying and selling equipment. Paco’s written offer had been prepared by a secretary who typed the total price as $10,900, rather than $109,000, which was the approximate fair market value of the equipment. Preston, on receipt of the offer, immediately accepted it. Paco learned of the error in the offer and refused to deliver the equipment to Preston unless Preston agreed to pay $109,000. Preston has sued Paco for breach of contract. Which of the following statements is correct?

  1. Paco will not be liable because there has been a mutual mistake of fact.
  2. Paco will be able to rescind the contract because Preston should have known that the price was erroneous.
  3. Preston will prevail because Paco is a merchant.
  4. The contract between Paco and Preston is void because the price set forth in the offer is substantially less than the equipment’s fair market value.
A

Paco will be able to rescind the contract because Preston should have known that the price was erroneous.

A contract based on a mistake is voidable by the party making the mistake if the mistake is so obvious that the other party should have known that a mistake was made. In this situation, Preston should have known that the $10,900 price stated in the offer was erroneous. Therefore, Paco may rescind the contract.

83
Q

On June 1, year 1, Nord Corp. engaged Milo & Co., CPAs, to perform certain management advisory services for 9 months for a $45,000 fee. The terms of their oral agreement required Milo to commence performance any time before October 1, year 1. On June 30, year 2, after Milo completed the work to Nord’s satisfaction, Nord paid Milo $30,000 by check. Nord conspicuously marked on the check that it constituted payment in full for all services rendered. Nord has refused to pay the remaining $15,000 arguing that, although it believes the $45,000 fee is reasonable, it had received bids of $30,000 and $38,000 from other firms to perform the same services as Milo. Milo endorsed and deposited the check. If Milo commences an action against Nord for the remaining $15,000, Milo will be entitled to recover

  1. $0 because there has been an enforceable accord and satisfaction.
  2. $0 because the Statute of Frauds has not been satisfied.
  3. $8,000 because $38,000 was the highest other bid.
  4. $15,000 because it is the balance due under the agreement.
A

$15,000 because it is the balance due under the agreement.

A payment of less than the originally agreed-upon price will serve as complete discharge of an obligation under a contract only if there is a bona fide dispute as to the amount owed, and both parties agree to the lesser sum. In this case Milo and Nord had agreed upon a fee for services of $45,000; thus, this debt was liquidated since the amount was certain. And since Milo had completed the work to Nord’s satisfaction, no bona fide dispute as to the amount owed arose; thus the debt remains liquidated, and Nord will be required to pay the full $45,000. The fact that Milo had endorsed and deposited the check which Nord had conspicuously marked as constituting payment in full is of no effect since the obligation was a liquidated debt, instead of an unliquidated debt.

84
Q

John Dash, an accountant, entered into a written contract with Kay Reese to perform certain tax services for Reese. Shortly thereafter, Reese was assessed additional taxes and she wanted to appeal the assessment. Reese was required to appeal immediately and the workpapers held by Dash were necessary to appeal. Dash refused to furnish Reese with the workpapers unless he was paid a substantially higher fee than was set forth in the contract. Reese reluctantly agreed in order to meet the filing deadline. The contract as revised is

  1. Voidable at Reese’s option based on undue influence.
  2. Voidable at Reese’s option based on duress.
  3. Void on the ground of undue influence.
  4. Void on the ground of duress.
A

Voidable at Reese’s option based on duress.

Ordinary duress is the actual or threatened causing of an action or inaction which, contrary to a party’s free will and judgment, forces him/her to enter into a contract. In this case, Dash’s retention of workpapers caused Reese to pay the higher-than-contracted price against her free will; thus the contract is voidable at Reese’s option based on ordinary duress.

Undue influence involves the forcing of a party to enter into a contract by taking unfair advantage of a relationship of trust between the two parties or by taking unfair advantage of another party’s weakness of mind.

85
Q

Race entered into a written agreement to sell a parcel of land to Lark for $50,000. At the time the agreement was executed, Race had consumed a large amount of alcoholic beverages which significantly impaired Race’s ability to understand the nature and terms of the contract. Lark knew Race was very intoxicated and that the land had been appraised at $95,000. Race wishes to void the contract. The contract is

  1. Voidable at Race’s option.
  2. Voidable at Race’s option only if the intoxication was involuntary.
  3. Void.
  4. Legally binding on both parties in the absence of fraud or undue influence.
A

Voidable at Race’s option.

Contracts are enforceable against a drunkard unless the drunkard was so intoxicated at the time of contracting that s/he lacked the ability to understand the nature and terms of the contract. A drunkard who lacked such ability has the option of voiding any contract entered into during the period of drunkeness, provided that s/he returns any item received under the terms of the contract in the same or similar condition. In this question, since it states that Race’s ability to understand was significantly impaired by his drunkenness, Race may void the contract at his option.

86
Q

Sable Corp. has $500,000 of outstanding accounts receivable. On July 10, year 1, Sable assigned a $50,000 account receivable due from Baker, one of Sable’s customers, to Rich Bank for value. On July 15, Baker paid Sable the $50,000. On July 17, Rich notified Baker of the July 10 assignment from Sable to Rich. Rich is entitled to collect $50,000 from

  1. Sable only.
  2. Baker only.
  3. Neither Sable nor Baker.
  4. Either Sable or Baker.
A

Sable only.

A debtor who does not have knowledge of a creditor’s assignment of his/her right to receive payment can extinguish all of his/her liability regarding the debt by paying the assignor. It is the duty of the assignee to notify the debtor of the assignment. In addition, an assignor who receives payment from the debtor after the assignment has occurred will always be liable to the assignee for payment received. In this case, since Baker paid the debt owed to Sable prior to receiving notice of the assignment, he/she has extinguished all liability on the debt. However, Sable is liable to Rich for payment received. Consequently, Rich is entitled to collect $50,000 from Sable only.

87
Q

Marglow Supplies, Inc. mailed a letter to Wilson Distributors on September 15, offering a 3-year franchise dealership. The offer stated the terms in detail and at the bottom stated that the offer would not be withdrawn prior to October 1. Which of the following is correct?

  1. The Statute of Frauds would not apply to the proposed contract.
  2. The offer is an irrevocable option which cannot be withdrawn prior to October 1.
  3. The offer cannot be assigned to another party by Wilson if Wilson chooses not to accept.
  4. A letter of acceptance from Wilson to Marglow sent on October 1, but not received until October 2, would not create a valid contract.
A

The offer cannot be assigned to another party by Wilson if Wilson chooses not to accept.

Offers to contract may only be accepted by the person to whom they were made. Offers are not assignable to others unless the offeror consents to such assignment.

B is incorrect as an option is only created when the offeree gives something of value to keep the offer open for a stated period of time. Therefore, no option was created, and Marglow can revoke its offer any time prior to acceptance by Wilson.

88
Q

Quick Corp. has $270,000 of outstanding accounts receivable. On March 10, Quick assigned a $30,000 account receivable due from Pine, one of Quick’s customers, to Taft Bank for value. On March 30, Pine paid Quick the $30,000. On April 5, Taft notified Pine of the March 10 assignment from Quick to Taft. Taft is entitled to collect $30,000 from

  1. Either Quick or Pine.
  2. Neither Quick nor Pine.
  3. Pine only.
  4. Quick only.
A

Quick only.

A debtor who does not have knowledge of a creditor’s assignment of his/her right to receive payment can extinguish all of his/her liability regarding the debt by paying the assignor. It is the duty of the assignee to notify the debtor of the assignment. In addition, an assignor who receives payment from the debtor after the assignment has occurred will always be liable to the assignee for payment received. In this case, since Pine paid the debt owed to Quick prior to receiving notice of the assignment, he/she has extinguished all liability on the debt. However, Quick is liable to Taft for payment received. Consequently, Taft is entitled to collect $30,000 from Quick only.

89
Q

Parent Corp. and Subsidiary Corp. file consolidated returns on a calendar-year basis. In January 2014, Subsidiary sold land, which it had used in its operations, to Parent for $75,000. Immediately before this sale, Subsidiary’s basis for the land was $45,000. Parent held the land primarily for sale to customers in the ordinary course of business. In July 2015, Parent sold the land to Dubin, an unrelated individual, for $90,000. In determining the consolidated taxable income for 2015, how much should Subsidiary take into account as a result of the 2014 sale of land from Subsidiary to Parent?

  1. $45,000
  2. $30,000
  3. $22,500
  4. $15,000
A

$30,000

Since Parent and Subsidiary file consolidated returns, the $30,000 gain ($75,000 — $45,000) realized by Subsidiary in 2014 would not have been recognized. Instead, the gain is deferred until 2015 when Parent sells the land to an unrelated party. Therefore, in 2015 Subsidiary would have a recognized gain of $30,000 and Parent would have a recognized gain of $15,000 ($90,000 — $75,000).

90
Q

Card communicated an offer to sell Card’s stereo to Bend for $250. Which of the following statements is correct regarding the effect of the communication of the offer?

  1. Bend should immediately accept or reject the offer to avoid liability to Card.
  2. Card is not obligated to sell the stereo to Bend until Bend accepts the offer.
  3. Card is required to mitigate any loss Card would sustain in the event Bend rejects the offer.
  4. Bend may not reject the offer for a reasonable period of time.
A

Card is not obligated to sell the stereo to Bend until Bend accepts the offer.

There is no contact until the offeree accepts the offer.

Acceptance:

  • May be written or oral
  • Offer may be accepted only by person to whom it was directed
  • Offeree must have knowledge of offer in order to accept
  • Intent to accept is required
  • Acceptance must generally be in form specified by offer
  • Acceptance must be unequivocal and unconditional (mirror image rule) under common law
  • Silence is not acceptance unless
  1. Offer indicated silence would constitute acceptance (e.g., offer states “your silence is accep­tance,” and offeree intended his/her silence as acceptance)
  2. Offeree has taken benefit of services or goods and exercised control over them when s/he had opportunity to reject them
91
Q

Samm, a plumber, entered into a contract for $75,000 with Orr, Inc. to perform certain plumbing services in a building owned by Orr. After Samm had satisfactorily performed the work, Orr discovered that Samm had violated the state licensing statute by failing to obtain a plumbing license. The licensing statute was enacted merely to raise revenue for the state. An independent appraisal of Samm’s work indicated the building’s fair market value increased by $70,000 as a result of Samm’s work. The cost of the materials which Samm supplied was $35,000. If Samm sues Orr, Samm will be entitled to recover

  • $0
  • $35,000
  • $70,000
  • $75,000
A

$75,000

In general, an agreement that violates a statute is illegal, making such agreement unenforceable. However, if the statute in question is a revenue-seeking licensing statute (in contrast to a regulatory licensing statute), the agreement would still be enforceable. This is an exception to the general rule. The plumbing licensing statute is a revenue-seeking licensing statute; thus, the contract violating the statute will still be enforceable for its full amount of $75,000.

92
Q

In deciding whether consideration necessary to form a contract exists, a court must determine whether

  1. The consideration given by each party is of roughly equal value.
  2. There is mutuality of consideration.
  3. The consideration has sufficient monetary value.
  4. The consideration conforms to the subjective intent of the parties.
A

There is mutuality of consideration.

The existence of consideration requires that there be mutuality of consideration (i.e., that each party is bound to give consideration in exchange for the consideration being given by the other party to the contract).

Mutuality of obligation—means both parties must be bound or neither is bound

93
Q

On June 30, 2015, Ral Corporation had retained earnings of $100,000. On that date, it sold a plot of land to a noncorporate stockholder for $50,000. Ral had paid $40,000 for the land in 2010, and it had a fair market value of $80,000 when the stockholder bought it. The amount of dividend income taxable to the stockholder in 2015 is

  1. $0
  2. $10,000
  3. $20,000
  4. $30,000
A

$30,000

If a corporation sells property to a shareholder for less than fair market value, the shareholder is considered to have received a constructive dividend to the extent of the difference between the fair market value of the property and the price paid. Thus, the shareholder’s dividend income is $30,000 ($80,000 FMV — $50,000 purchase price).

94
Q

John Tuck entered into a contract with Jack Doe. Doe asserts that he entered into the contract under duress. Which of the following best describes a necessary element of duress?

  1. There must have been a confidential or fiduciary relationship between Tuck and Doe.
  2. The contract entered into between Tuck and Doe was unconscionable.
  3. Doe entered into the contract with Tuck because of Tuck’s improper threats.
  4. Tuck must have intended that Doe be influenced by the improper threats.
A

Doe entered into the contract with Tuck because of Tuck’s improper threats.

Duress exists when one of the contracting parties induces the other party into the agreement by acting or threatening to act in such a manner so as to deprive the second party of his free will in making the decision to enter the contract. Doe can therefore claim duress if improper threats are made by Tuck which deprive Doe of his free will and cause Doe to agree.

95
Q

Diel entered into a written contract to sell a building to Stone. The contract was properly recorded. Stone breached the contract and Diel has brought an action for breach of contract. Stone pleads the statute of limitations as a defense. The

  1. time period fixed by the statute of limitations is uniform throughout the states.
  2. recording of the contract stops the running of the statute of limitations.
  3. time period fixed by the statute of limitations begins when the contract is recorded.
  4. remedy sought by Diel will be barred when the period of time provided by the statute of limitations has expired.
A

Remedy sought by Diel will be barred when the period of time provided by the statute of limitations has expired.

Statute of limitations bars action at law on contracts unless they are brought within prescribed periods of time. In this case, if Diel did not bring an action against Stone before the statute of limitations had expired, the court would rule in favor of Stone and would not allow Diel to pursue the action further.

96
Q

Foster offered to sell Lebow his garage for $27,000. The offer was in writing and signed by Foster. Foster gave Lebow 5 days to decide. On the fourth day Foster accepted a better offer from Dilby, who was unaware of the offer to Lebow. Foster subsequently conveyed the property to Dilby. Unaware of the sale to Dilby, Lebow telephoned Foster on the fifth day and unconditionally accepted the offer. Under the circumstances, Lebow

  1. Is entitled to specific performance by Foster.
  2. Has no rights against Foster.
  3. Is entitled to damages.
  4. Can obtain specific performance by Dilby upon depositing in court the $27,000 he agreed to pay.
A

Is entitled to damages.

Lebow’s acceptance created a contract due to the fact Foster had not revoked the offer prior to acceptance. Since the offer did not qualify as a firm offer (i.e., did not involve the sale of goods) or an option (i.e., no consideration was given), Foster had the power to revoke the offer. However, a revocation must be communicated to the offeree to terminate the offer. Consequently, the offer was in existence when accepted by Lebow.

97
Q

On May 1, Dix and Wilk entered into an oral agreement by which Dix agreed to purchase a small parcel of land from Wilk for $450. Dix paid Wilk $100 as a deposit. The following day, Wilk received another offer to purchase the land for $650, the fair market value. Wilk immediately notified Dix that Wilk would not sell the land for $450. If Dix sues Wilk for specific performance, Dix will

  1. Prevail, because the amount of the contract was less than $500.
  2. Prevail, because there was part performance.
  3. Lose, because the fair market value of the land is over $500.
  4. Lose, because the agreement was not in writing and signed by Wilk.
A

Lose, because the agreement was not in writing and signed by Wilk.

In order to conform with the Statute of Frauds, and therefore be enforceable, any contract for the sale of an interest in real property (regardless of price) must be in writing and signed by the party to be charged. Thus, in this case, Dix will lose because the agreement was not in writing and signed by Wilk.

98
Q

Green purchased a business from Tanner. During negotiations, Tanner told Green that it was a wonderful business that Green will love. He also told Green that he was sure she will make at least $150,000 next year. At the end of the next year, it turns out that Green only makes a profit of $75,000, even though she worked very hard putting in 90 to 100 hours per week. She sues Tanner for fraud. Who wins?

  1. Green, because she earned half of what Tanner had said.
  2. Green, because she was told that she would love the business and is now dissatisfied.
  3. Tanner, because none of the statements constitute actionable fraud.
  4. Tanner, because no matter what Green is told to encourage her to purchase a business, she must realize that it is “buyer beware.”
A

Tanner, because none of the statements constitute actionable fraud.

One of the elements needed to establish fraud is a misrepresentation of a material fact. When Tanner told Green that she will love the business, this is sales talk and at best a prediction, not a fact. Tanner’s statement that he is sure she will make at least $150,000 is a prediction not a fact.

99
Q

Moss Corp.’s income statement for the current year showed the following expenses for life insurance premiums:

  • Group-term life insurance premiums paid on employees’ lives with the employees’ dependents as beneficiaries $10,000
  • Term life insurance premiums paid on life of Moss’ president with Moss Corp. as beneficiary $7,000

On its current year tax return, how much should Moss deduct for life insurance premiums?

  1. $0
  2. $7,000
  3. $10,000
  4. $17,000
A

$10,000

Group-term life insurance premiums paid by an employer for policies on the lives of its employees, with the employees’ dependents as beneficiaries are always deductible by the employer. Life insurance premiums on the life of a corporate officer are nondeductible when the corporation is the beneficiary. The premiums are nondeductible because the proceeds paid to the corporation upon the death of the officer will be nontaxable. Therefore, deductible life insurance premiums total $10,000.

100
Q

Johns leased an apartment from Olsen. Shortly before the lease expired, Olsen threatened Johns with eviction and physical harm if Johns did not sign a new lease for twice the old rent. Johns, unable to afford the expense to fight eviction, and in fear of physical harm, signed the new lease. Three months later, Johns moved and sued to void the lease claiming duress. The lease will be held

  1. Void because of the unreasonable increase in rent.
  2. Voidable because of Olsen’s threat to bring eviction proceedings.
  3. Void because of Johns’ financial condition.
  4. Voidable because of Olsen’s threat of physical harm.
A

Voidable because of Olsen’s threat of physical harm.

Olsen’s threat of physical harm would constitute duress. Duress is any act, threat of violence, or extreme pressure against a party which deprives that party of free will and causes him/her to agree. Duress causes an agreement to be voidable.

101
Q

Beal offered in writing to sell Crane a parcel of land for $150,000. If Beal dies, the offer will

  1. Automatically terminate prior to Crane’s acceptance.
  2. Automatically terminate despite Crane’s prior acceptance.
  3. Terminate prior to Crane’s acceptance only if Crane received notice of Beal’s death.
  4. Remain open for a reasonable period of time after Beal’s death.
A

Automatically terminate prior to Crane’s acceptance.

The death of an offeror automatically and immediately terminates an offer to contract. This is true regardless of whether the offeree has received notice of the death. However, if acceptance occurs prior to the death of the offeror, manifestation of mutual assent is present and it will be binding.

102
Q

Ace Corp. files a consolidated return with its wholly owned subsidiary, Barr Corp. During the current year Barr paid a cash dividend of $10,000 to Ace. How much of this dividend is taxable on the current year consolidated return?

  1. $0
  2. $1,500
  3. $8,500
  4. $10,000
A

$0

A dividend distribution from one member to another member of an affiliated group during a consolidated return year is eliminated in determining consolidated taxable income. Note, that if a consolidated return is not filed, dividends received from an affiliated corporation (80% or more ownership) are eligible for a 100% dividends-received deduction.

103
Q

Stone engaged Parker to perform personal services for $1,000 a month for a period of 3 months. The contract was entered into orally on August 1, year 1, and performance was to commence on January 1, year 2. On September 15, year 1, Parker anticipatorily repudiated the contract. As a result, Stone can

  1. Obtain specific performance.
  2. Not assign her rights to damages under the contract to a third party.
  3. Immediately sue for breach of contract.
  4. Not enforce the contract against Parker since the contract is oral.
A

Immediately sue for breach of contract.

When Parker anticipatorily repudiated a contract, Stone could immediately sue for breach of contract or wait for performance on the appointed date, and then sue for breach if performance was not rendered.

Note that specific performance is not available as a remedy for breach of a personal service contract. It would violate the constitutional amendment prohibiting involuntary servitude.

104
Q

Aqua, Inc., a Florida corporation, entered into a contract for $30,000 with Sing, Inc., to perform plumbing services in a complex owned by Sing in Virginia. After the work was satisfactorily completed, Sing discovered that Aqua violated Virginia’s licensing law by failing to obtain a plumbing license. Virginia’s licensing statute was regulatory in nature, serving to protect the public against unskilled and dishonest plumbers. Upon Sing’s request, independent appraisals of Aqua’s work were performed, which indicated that the complex was benefited to the extent of $25,000. Sing refuses to pay Aqua. If Aqua brings suit it may recover

  1. $30,000.
  2. $25,000.
  3. Nothing.
  4. An amount sufficient to cover its out-of-pocket costs.
A

Nothing

If a contract violates a regulatory licensing statute, it will be unenforceable by either party. The main function of a regulatory licensing statute is to protect the public against unskilled or dishonest persons. Another type of licensing statute is a revenue-seeking statute. The purpose of these types of statutes generally is to gain revenue for the governmental unit issuing the license. A contract that violates a revenue-seeking statute is enforceable. In this case, the facts stipulate that a regulatory licensing statute is violated. Consequently, Aqua may not enforce the agreement.

105
Q

Wren purchased a factory from First Federal Realty. Wren paid 20% at the closing and gave a note for the balance secured by a 20-year mortgage. Five years later, Wren found it increasingly difficult to make payments on the note and defaulted. First Federal threatened to accelerate the loan and foreclose if Wren continued in default. First Federal told Wren to make payment or obtain an acceptable third party to assume the obligation. Wren offered the land to Moss, Inc. for $10,000 less than the equity Wren had in the property. This was acceptable to First Federal and at the closing Moss paid the arrearage, assumed the mortgage and note, and had title transferred to its name. First Federal released Wren. The transaction in question is a(n)

  1. Purchase of land subject to a mortgage.
  2. Assignment and delegation.
  3. Third-party beneficiary contract.
  4. Novation.
A

Novation.

A novation is an agreement between three parties whereby a previous agreement is discharged by the creation of a new agreement. The new agreement substitutes one contracting party for another. In this case, Moss was substituted for Wren, and Wren was discharged of all liability.

106
Q

On August 1, Neptune Fisheries contracted in writing with West Markets to deliver to West 3,000 pounds of lobsters at $4.00 a pound. Delivery of the lobsters was due October 1 with payment due November 1. On August 4, Neptune entered into a contract with Deep Sea Lobster Farms which provided as follows: “Neptune Fisheries assigns all the rights under the contract with West Markets dated August 1 to Deep Sea Lobster Farms.” The best interpretation of the August 4 contract would be that it was

  1. Only an assignment of rights by Neptune.
  2. Only a delegation of duties by Neptune.
  3. An assignment of rights and a delegation of duties by Neptune.
  4. An unenforceable third-party beneficiary contract.
A

An assignment of rights and a delegation of duties by Neptune.

An assignment of a contractual agreement is taken to mean both the assignment of the rights of the contract as well as the delegation of the duties of the contract. In this case, Neptune has assigned the contract to Deep Sea Lobster.

107
Q

Meek & Co., CPAs, was engaged by Reed, the president of Sulk Corp., to issue by June 15, year 1, an opinion on Sulk’s financial statements for the fiscal year ended March 31, year 1. Meek’s engagement and its fee of $20,000 were approved by Sulk’s board of directors. Meek did not issue its opinion until June 30 because of Sulk’s failure to supply Meek with the necessary information to complete the audit. Sulk refuses to pay Meek. If Meek sues Sulk, Meek will

  1. Prevail based on the contract.
  2. Prevail based on the quasi contract.
  3. Lose, since it breached the contract.
  4. Lose, since the June 15 deadline was a condition precedent to Sulk’s performance.
A

Prevail based on the contract.

Meek & Co., CPAs, and Sulk Corp. have entered into a valid contract which required that Meek issue an opinion by June 15, year 1. However, Meek did not breach its contract due to failure to perform by the agreed-upon date because the alleged breach was caused by Sulk’s failure to provide Meek with the necessary information. In every contract there is an implied agreement that the parties to the contract will not interfere with or hinder the other party’s performance.

A quasi contract is an implied-in-law rather than express agreement which results when one of the parties has been unjustly enriched at the expense of the other. The law creates such a contract when there is no binding agreement present to keep the unjust enrichment from occurring. In this case, Meek and Sulk have entered into a valid contract which would be binding on both parties.

108
Q

Rand Corporation and Hardy Corporation are attempting to create a contract over the Internet in which Rand Corporation will sell some land to Hardy Corporation for $850,000. Which of the following statements is correct regarding this attempt to make this contract?

  1. The Statute of Frauds does not allow this contract to be made over the Internet.
  2. Agents authorized by Rand Corporation and Hardy Corporation must meet together to sign this contract in person.
  3. This contract can be completed over the Internet.
  4. Only if this contract did not involve real estate could it be accomplished over the Internet.
A

This contract can be completed over the Internet.

The issue of allowing signatures over the Internet has been addressed at both the federal and state levels. A majority of states have now passed laws to allow electronic signatures and invite technology to determine their validity.

109
Q

Baxter agreed in writing to sell Faxton some land for $150,000. A few days after both signed the contract, Baxter said he would not sell Faxton the land unless she agreed to pay an additional $10,000. She agreed in writing and received title and possession of the land after paying $160,000. She later brings suit to get back the $10,000. Does Faxton win the suit?

  1. Yes, because the original contract was for $150,000.
  2. Yes, if she brings suit within a statutory cooling off period.
  3. No, because she agreed to the additional $10,000.
  4. No, because there was no duress involved in this fact pattern.
A

Yes, because the original contract was for $150,000.

Under common law, modifications of contracts require new consideration on both sides. Since the subject matter of this contract is land, common law does apply. Even though Faxton gave new consideration by agreeing to pay the extra $10,000, Baxter did not give new consideration.

110
Q

On July 1, Silk, Inc. sent Blue a telegram offering to sell Blue a building for $80,000. In the telegram, Silk stated that it would give Blue 30 days to accept the offer. On July 15, Blue sent Silk a telegram that included the following statement: “The price for your building seems too high. Would you consider taking $75,000?” This telegram was received by Silk on July 16. On July 19, Tint made an offer to Silk to purchase the building for $82,000. Upon learning of Tint’s offer, Blue, on July 27, sent Silk a signed letter agreeing to purchase the building for $80,000. This letter was received by Silk on July 29. However, Silk now refuses to sell Blue the building. If Blue commences an action against Silk for breach of contract, Blue will

  1. Win, because Blue effectively accepted Silk’s offer of July 1.
  2. Win, because Silk was obligated to keep the offer open for the 30-day period.
  3. Lose, because Blue sent the July 15 telegram.
  4. Lose, because Blue used an unauthorized means of communication.
A

Win, because Blue effectively accepted Silk’s offer of July 1.

Blue effectively accepted Silk’s offer to sell prior to the termination of the offer. Normally, a counteroffer terminates an offer when communicated to the offeror. However, Blue’s inquiry into the selling price on July 15 did not constitute a counteroffer since it was merely a request for different terms. Thus, the offer remained open beyond July 15. Also, Blue’s knowledge of Tint’s offer to purchase the building did not serve as a revocation of Silk’s offer. Therefore, the offer to sell the building remained open until acceptance by Blue on July 29.